Knewton inference 2

This topic has expert replies
User avatar
Master | Next Rank: 500 Posts
Posts: 150
Joined: Tue Aug 18, 2009 7:36 am
Location: gurgaon
Thanked: 8 times
Followed by:2 members

Knewton inference 2

by itsmebharat » Wed Jun 01, 2011 12:26 am
The number of workers diagnosed with a certain respiratory condition at a small processing plant has dropped significantly in the last year. The factory management attributes this decrease entirely to the new dust control systems, which drastically improve the air quality in the factory.

Which of the following is evidence cited in this argument?

* (A)

The number of workers diagnosed with a respiratory condition at a small processing plant has dropped in the last year.
* (B)

A new diagnostic procedure shows that many factory workers who until this year were thought to have the respiratory condition actually suffer from another condition.
* (C)

There is a relationship between the installation of dust control systems and the occurence of a certain respiratory condition.
* (D)

The new dust control systems are state of the art.
* (E)

The decrease in respiratory condition diagnoses is entirely because of the new dust control systems.

OA A
I am not an Expert, please feel free to suggest if there is an error.

User avatar
Master | Next Rank: 500 Posts
Posts: 439
Joined: Sat Aug 09, 2008 8:32 am
Location: India
Thanked: 34 times
Followed by:28 members

by sivaelectric » Wed Jun 01, 2011 1:08 am
Choice is A
If I am wrong correct me :), If my post helped let me know by clicking the Thanks button ;).

Chitra Sivasankar Arunagiri

User avatar
Legendary Member
Posts: 1309
Joined: Mon Apr 04, 2011 5:34 am
Location: India
Thanked: 310 times
Followed by:123 members
GMAT Score:750

by cans » Wed Jun 01, 2011 1:29 am
IMO A

User avatar
Legendary Member
Posts: 979
Joined: Tue Apr 14, 2009 1:38 am
Location: Hyderabad, India
Thanked: 49 times
Followed by:12 members
GMAT Score:700

by bubbliiiiiiii » Wed Jun 01, 2011 1:31 am
Why not C?
Regards,

Pranay

User avatar
Legendary Member
Posts: 1309
Joined: Mon Apr 04, 2011 5:34 am
Location: India
Thanked: 310 times
Followed by:123 members
GMAT Score:750

by cans » Wed Jun 01, 2011 4:32 am
bubbliiiiiiii wrote:Why not C?
Which of the following is evidence cited in this argument?
we have to find the evidence stated.
C)is what factory management thinks.
A) is the fact (as given in 1st line and thus evidence)

Senior | Next Rank: 100 Posts
Posts: 43
Joined: Fri Apr 08, 2011 7:32 am
Thanked: 2 times

by subhashghosh » Wed Jun 01, 2011 4:43 am
+1 for A.

Master | Next Rank: 500 Posts
Posts: 112
Joined: Sun Nov 08, 2009 9:39 pm
Location: Delhi
Thanked: 2 times

by dv2020 » Wed Jun 01, 2011 7:01 am
itsmebharat wrote:The number of workers diagnosed with a certain respiratory condition at a small processing plant has dropped significantly in the last year. The factory management attributes this decrease entirely to the new dust control systems, which drastically improve the air quality in the factory.

Which of the following is evidence cited in this argument?

* (A)

The number of workers diagnosed with a respiratory condition at a small processing plant has dropped in the last year.
* (B)

A new diagnostic procedure shows that many factory workers who until this year were thought to have the respiratory condition actually suffer from another condition.
* (C)

There is a relationship between the installation of dust control systems and the occurence of a certain respiratory condition.
* (D)

The new dust control systems are state of the art.
* (E)

The decrease in respiratory condition diagnoses is entirely because of the new dust control systems.

OA A
The words certain and significantly have been dropped from the the first sentence of the argument, which is clearly the evidence. I think significantly makes a lot of difference in the context of the argument. What if the number of workers themselves dropped for someother reason. For eg change in a method of production. Its only the use of significantly in the evidence which makes us believe that it can act as a evidence for some method of prevention from the breathing disorder becoming sucessful as mentioned in the next sentence. Hence I picked C. I am not convinced with the OA
Want to hear from others.....

User avatar
Senior | Next Rank: 100 Posts
Posts: 99
Joined: Sat Oct 16, 2010 1:55 pm
Thanked: 1 times

by aftableo2006 » Wed Jun 01, 2011 7:47 am
it is A simple one

User avatar
Legendary Member
Posts: 1309
Joined: Mon Apr 04, 2011 5:34 am
Location: India
Thanked: 310 times
Followed by:123 members
GMAT Score:750

by cans » Wed Jun 01, 2011 7:48 am
dv2020 wrote:The words certain and significantly have been dropped from the the first sentence of the argument, which is clearly the evidence. I think significantly makes a lot of difference in the context of the argument. What if the number of workers themselves dropped for someother reason. For eg change in a method of production. Its only the use of significantly in the evidence which makes us believe that it can act as a evidence for some method of prevention from the breathing disorder becoming sucessful as mentioned in the next sentence. Hence I picked C. I am not convinced with the OA
Want to hear from others.....
We are not concerned about the reason why no. of people diagnosed are less.
The question asks for evidence given in argument. not the cause of the evidence.

Master | Next Rank: 500 Posts
Posts: 112
Joined: Sun Nov 08, 2009 9:39 pm
Location: Delhi
Thanked: 2 times

by dv2020 » Wed Jun 01, 2011 7:58 am
My question is simple :

the number dropped significantly

the number dropped

which one will you pick when the argument clearly says number dropped significantly

User avatar
Legendary Member
Posts: 1309
Joined: Mon Apr 04, 2011 5:34 am
Location: India
Thanked: 310 times
Followed by:123 members
GMAT Score:750

by cans » Wed Jun 01, 2011 5:43 pm
dv2020 wrote:My question is simple :

the number dropped significantly

the number dropped

which one will you pick when the argument clearly says number dropped significantly
Well the evidence is number dropped significantly. say from 1000 to 300
but "the number dropped" is also correct ( as in this case we are not measuring the extent)
What's the answer then according to you?

Junior | Next Rank: 30 Posts
Posts: 13
Joined: Sat May 07, 2011 2:10 pm

by shrivast » Wed Jun 01, 2011 7:25 pm
I am still not sure about A, the evidence is required to prove that "The decrease in respiratory condition diagnoses is entirely because of the new dust control systems", as per A the decrease could be due to lot of other reasons and not just the new dust control systems...

User avatar
Legendary Member
Posts: 1325
Joined: Sun Nov 01, 2009 6:24 am
Thanked: 105 times
Followed by:14 members

by vikram4689 » Wed Jun 01, 2011 10:15 pm
Earlier even i thought the same way, word "evidence" stimulates the mind to think for an option that provides the basis on which the conclusion can be based on. But be careful that would go into ASSUMPTION Q. and answer would be C for that Q. Here ques. is asking for "evidence cited" which i learned is equivalent to STATED FACT. Though old question. type but learned a new representation of the ques. Thanks.
Premise: If you like my post
Conclusion : Press the Thanks Button ;)

User avatar
Master | Next Rank: 500 Posts
Posts: 150
Joined: Tue Aug 18, 2009 7:36 am
Location: gurgaon
Thanked: 8 times
Followed by:2 members

by itsmebharat » Wed Jun 01, 2011 10:35 pm
If you read the ques stem again, it asks for evidence cited in this argument. lets first see the argument

The number of workers diagnosed ... has dropped ... in the last year.
... this decrease entirely to the new dust control systems, ....

Option B talks about another diagnostic procedure, Doesnt that goes out of the scope from the argument.

(A)The number of workers diagnosed with a respiratory condition at a small processing plant has dropped in the last year.
(B)A new diagnostic procedure shows that many factory workers who until this year were thought to have the respiratory condition actually suffer from another condition. out of scope
(C)There is a relationship between the installation of dust control systems and the occurence of a certain respiratory condition. irrelevant
(D)The new dust control systems are state of the art. irrelevant
(E)The decrease in respiratory condition diagnoses is entirely because of the new dust control systems. its already stated in the argument "this decrease entirely to the new dust control systems"
I am not an Expert, please feel free to suggest if there is an error.

User avatar
Legendary Member
Posts: 1309
Joined: Mon Apr 04, 2011 5:34 am
Location: India
Thanked: 310 times
Followed by:123 members
GMAT Score:750

by cans » Thu Jun 02, 2011 3:12 am
https://www.beatthegmat.com/knewton-infe ... tml#371296
A similar question but instead of evidence cited, assumption is asked.